LSAT and Law School Admissions Forum

Get expert LSAT preparation and law school admissions advice from PowerScore Test Preparation.

 Administrator
PowerScore Staff
  • PowerScore Staff
  • Posts: 8916
  • Joined: Feb 02, 2011
|
#32467
Complete Question Explanation

Flaw in the Reasoning. The correct answer choice is (C)

Here, the mayor dismisses concerns raised by protestors about the proposed construction of a chemical plant. Although a large protest was staged in the town to oppose the factory’s construction, most of the protestors were paid by property developers worried about the effect the construction would have on the value of their land near the proposed build site. Since the only reason most of the protestors showed up was the fact that they were paid to do so, the mayor concludes that the protestors’ claim that the factory would lead to health problems can be dismissed.

In reaching this conclusion, the mayor made a Source Argument, focusing on the paid protestors’ motive for making their claim concerning potential health problems related to the factory, rather than addressing the substance of their claim.

The question stem tells us that this is a Flaw in the Reasoning question. Our prephrase is that the correct answer will describe the mayor’s Source Argument.

Answer choice (A): This refers to a Straw Man argument. However, the mayor did not mischaracterize the protestors’ argument.

Answer choice (B): This refers to an Appeal to Emotion. The mayor, however, did not try to induce fear in the reader.

Answer choice (C): This is the correct answer choice, because it describes the mayor’s Source Argument, in which the mayor rejected the claim regarding potential health problems related to the factory based on the fact that a source of the argument was protestors paid to participate in the protest.

Answer choice (D): This refers to an Overgeneralization from an Exceptional Case. But, the mayor did not reach a broad conclusion based on a small, and potentially unrepresentative, data source.

Answer choice (E): The mayor did not infer that a result was inevitable. Instead, the conclusion was that the protestors’ claim could be dismissed.
 c-erv
  • Posts: 15
  • Joined: Oct 14, 2016
|
#30496
Hello, I recognize C as the right answer for this problem however, I am having a hard time seeing why answer choice D is wrong.
In the stimulus above, the mayor claims that we shouldn't worry about the health problems that might be caused by the chemical plant because "MOST" of the protestors were paid off property developers. Would this not be an unrepresentative case? or is it wrong because of the term most vs few in the answer choice?

thanks
User avatar
 Jonathan Evans
PowerScore Staff
  • PowerScore Staff
  • Posts: 726
  • Joined: Jun 09, 2016
|
#30547
C-erv,

You have identified an excellent reason why (D) is incorrect ("most" of the protesters). That alone is sufficient to get rid of it. These are not a few, unrepresentative cases. Further, rely on your prephrase here. The most salient flaw is the "Source" argument; such arguments are relatively uncommon but should lead to strong prephrases when present. I would suggest revisiting the lesson about this common flaw as part of your review.

Get the most out of your LSAT Prep Plus subscription.

Analyze and track your performance with our Testing and Analytics Package.